Đến nội dung

Dam Uoc Mo nội dung

Có 424 mục bởi Dam Uoc Mo (Tìm giới hạn từ 20-04-2020)



Sắp theo                Sắp xếp  

#666188 Tìm GTLN của $\left ( k.n \right )$

Đã gửi bởi Dam Uoc Mo on 29-12-2016 - 20:52 trong Đa thức

Cho đa thức $P(x)$ thỏa mãn $\left\{\begin{matrix} P(x_{1})=P(x_{2})=...P(x_{k})=54; x_{i}\neq x_{j} \\\ P(y_{1})=P\left ( y_{2} \right )=...=P(y_{n})=2013; y_{i}\neq y_{j} \end{matrix}\right.$
Tìm GTLN của $\left ( k.n \right )$. với $x_{i},y_{j}$ nguyên.



#664583 CMR $lim(n.u_{n})=0$

Đã gửi bởi Dam Uoc Mo on 13-12-2016 - 22:01 trong Dãy số - Giới hạn

Cho dãy dương $(u_{n})$ bị chặn thỏa mãn $u_{n+1} \leq u_{n} + (u_{n})^{2}, \forall n \in \mathbb{N}.$ Chứng minh $lim(n.u_{n})=0$.




#664108 CMR: $limx_{n}=b$

Đã gửi bởi Dam Uoc Mo on 07-12-2016 - 22:15 trong Dãy số - Giới hạn

Cho dãy $\left ( x_{n} \right ): \left\{\begin{matrix}x_{0}=a \in R \\ x_{n+1}=sin^{2}\left ( x_{n}+3 \right )-2017 \end{matrix}\right.$

Gọi $b$ là nghiệm duy nhất của PT: $sin^{2}\left ( x+3 \right )-x=2017.$

CMR: $limx_{n}=b$.




#664028 CMR với mọi cách chọn $a,b$ nguyên thì dãy trên hoặc không có số nà...

Đã gửi bởi Dam Uoc Mo on 06-12-2016 - 22:25 trong Số học

Cho $(u_{n})$:
$u_{0}=a,u_{1}=b$ với $a,b$ nguyên.
$u_{n+1}=5(u_{n})^{2} -3u_{n-1}$.
CMR với mọi cách chọn $a,b$ nguyên thì dãy trên hoặc không có số nào chia hết cho $1997$ hoặc có vô số số chia hết cho $1997$.



#663829 CMR không tồn tại $f(x)$ hệ số nguyên có bậc nguyên dương thỏa mãn...

Đã gửi bởi Dam Uoc Mo on 04-12-2016 - 21:58 trong Số học

Bài 1: CMR không tồn tại $f(x)$ hệ số nguyên có bậc nguyên dương thỏa mãn $f(k)$ nguyên tố với mọi $k$ nguyên dương.

 

Bài 2: CMR với mọi $n$ nguyên dương,$n\geq 2$ thì luôn tồn tại $a,b$ nguyên dương sao cho $(a+i,b+j)>1$ với mọi $i;j\in \left \{ \left. 1;2;...;n-1 \right \} \right.$

 

Bài 3: Cho $f_{1}(x);f_{2}(x);...;f_{n}(x)$ là $n$ đa thức với hệ số nguyên khác $0$. CMR tồn tại $P(x)$ hệ số nguyên sao cho với mọi  $i=\overline{1;n}$ ta luôn có $P(x)+f_{i}(x)$ bất khả quy trên $\mathbb{Z}$.

 




#663597 $\sum \left | A_{i}\bigcap A_{j}...

Đã gửi bởi Dam Uoc Mo on 01-12-2016 - 23:30 trong Tổ hợp và rời rạc

Cho 1 tập có $n$ phần tử. Với 2 tập con $A_{i}$ và $A_{j}$ với $i\neq j$ ta tính $\left | A_{i}\bigcap A_{j} \right |$, sau đó đem cộng tất cả các kết quả đó lại với nhau được $S_{n}$. Tính $S_{n}$.

P/S: Bài này mình đi theo tư tưởng tuy hồi thì tính ra $S_{n+1}=4S_{n}+\frac{4^{n}-2^{n}}{2}$, song cái này chưa đúng do $S_{2}=2,S_{3}=16$, xem lại mãi vẫn không biết sai ở đâu. :/ 




#663243 $A=17^{2006n}+4.17^{2n}+7.19^{5n}$ kh...

Đã gửi bởi Dam Uoc Mo on 27-11-2016 - 22:37 trong Số học

Chứng minh rằng $A=17^{2006n}+4.17^{2n}+7.19^{5n}$ không thể phân tích thành tích 2 số nguyên dương liên tiếp.




#662861 $\sum_{cyc}a^{2}b^{2}(a+b)\leq 2...

Đã gửi bởi Dam Uoc Mo on 23-11-2016 - 23:02 trong Bất đẳng thức - Cực trị

Cho $a,b,c \geq 0;a+b+c=2$. CMR:
$a, \prod_{cyc}(a^{2}+ab+b^{2})\leq 3. \\ b, \sum_{cyc}a^{2}b^{2}(a+b)\leq 2.$
Câu $a$ mọi người đừng dồn biến nhé.



#661870 $1+2^{n}+4^{n}$ là số nguyên tố

Đã gửi bởi Dam Uoc Mo on 14-11-2016 - 11:22 trong Số học

Bài 1: $2^{p}+2^{q}\vdots (pq). (*)$

Trường hợp 1: $p=q$

 

Thay vào (*) ta được $p^2 | 2.2^p$ suy ra $p=2$.

 

Trường hợp 2: $ p \ne q$.

Trường hợp 2.1: Một trong hai số $p,q$ bằng 2. Không mất tính tổng quát giả sử $q=2, p \ne 2$ Khi đó $2p |2^2+2^p \Rightarrow 2^2+2^p \equiv 0 (mod p)$ Mặt khác theo định lý Fermat, ta có:

$2^p \equiv 2 (mod p)$ $\Rightarrow 0 \equiv 2^2+2=2.3 (mod p)$ nên $p=3$. Kiểm tra trực tiếp thấy thỏa mãn vậy $(2,3), (3,2)$ thỏa mãn bài toán.

Trường hợp 2.2: $p \ne q, p,q \ne 2$. Theo định lí Fermat ta có:

$0 \equiv 2^p+2^q \equiv 2+2^q \equiv 2(1+2^{q-1}) (mod p) \Rightarrow p | 2(2^{q-1}+1) \Rightarrow p | 1+ 2^{q-1} (1) \Rightarrow 2^{2(q-1)} \equiv 1 (mod p)$

 

$\Rightarrow ord_p(2)| 2(q-1) \Rightarrow ord_p(2) | (2(q-1),p-1)=2(q-1,p-1)$. Do $p,q$ là hai số nguyên tố lẻ đặt:

 

$p-1=2^lm,q-1=2^kn$ trong đó $m, n$ là các số lẻ $k, l >0$. Khi đó $ord_p(2) | 2^{k+1}$ Như vậy $ord_p(2)=2^h ( 0 \le h \le k+1)$

 

Nếu $ 0 \le h \le k$ thì:  $((2^{2^h})^{2^{k-h}})^m \equiv 1 (mod p)$ hay $2^{q-1}-1 \equiv 0 (mod p)$ Mâu thuẫn với (1).

 

Vậy $ord_p(2)=2^{k+1} \Rightarrow 2^{k+1} | \varphi (p)=p-1=2^lm \Rightarrow 2^{k+1}|2^l \Rightarrow k<l$. Tương tự ta chứng minh được $l < k$ (vô lí). 

 

Kết luận $(p,q)=(2,2), (2,3), (3,2)$ thỏa mãn bài toán

 

Bài tương tự: Tìm $p,q$ nguyên tố sao cho $5^p+5^q \vdots pq$

Bài 2: bạn có thể xem lại đề không mình chỉ chứng minh được $n$ có dạng $3^k$ :(

Cho hỏi lại đoạn này tí, nhỡ nó chứa cả ước lẻ thì sao ?

Còn bài 2 liệu đặt n như thế có được không ?Nhỡ nó có ước nguyên tố khác thì sao? Mình cũng đọc có sách ghi vậy nên thắc mắc.




#661845 $1+2^{n}+4^{n}$ là số nguyên tố

Đã gửi bởi Dam Uoc Mo on 13-11-2016 - 23:30 trong Số học

Bài 1: $2^{p}+2^{q}\vdots (pq). (*)$

Trường hợp 1: $p=q$

 

Thay vào (*) ta được $p^2 | 2.2^p$ suy ra $p=2$.

 

Trường hợp 2: $ p \ne q$.

Trường hợp 2.1: Một trong hai số $p,q$ bằng 2. Không mất tính tổng quát giả sử $q=2, p \ne 2$ Khi đó $2p |2^2+2^p \Rightarrow 2^2+2^p \equiv 0 (mod p)$ Mặt khác theo định lý Fermat, ta có:

$2^p \equiv 2 (mod p)$ $\Rightarrow 0 \equiv 2^2+2=2.3 (mod p)$ nên $p=3$. Kiểm tra trực tiếp thấy thỏa mãn vậy $(2,3), (3,2)$ thỏa mãn bài toán.

Trường hợp 2.2: $p \ne q, p,q \ne 2$. Theo định lí Fermat ta có:

$0 \equiv 2^p+2^q \equiv 2+2^q \equiv 2(1+2^{q-1}) (mod p) \Rightarrow p | 2(2^{q-1}+1) \Rightarrow p | 1+ 2^{q-1} (1) \Rightarrow 2^{2(q-1)} \equiv 1 (mod p)$

 

$\Rightarrow ord_p(2)| 2(q-1) \Rightarrow ord_p(2) | (2(q-1),p-1)=2(q-1,p-1)$. Do $p,q$ là hai số nguyên tố lẻ đặt:

 

$p-1=2^lm,q-1=2^kn$ trong đó $m, n$ là các số lẻ $k, l >0$. Khi đó $ord_p(2) | 2^{k+1}$ Như vậy $ord_p(2)=2^h ( 0 \le h \le k+1)$

 

Nếu $ 0 \le h \le k$ thì:  $((2^{2^h})^{2^{k-h}})^m \equiv 1 (mod p)$ hay $2^{q-1}-1 \equiv 0 (mod p)$ Mâu thuẫn với (1).

 

Vậy $ord_p(2)=2^{k+1} \Rightarrow 2^{k+1} | \varphi (p)=p-1=2^lm \Rightarrow 2^{k+1}|2^l \Rightarrow k<l$. Tương tự ta chứng minh được $l < k$ (vô lí). 

 

Kết luận $(p,q)=(2,2), (2,3), (3,2)$ thỏa mãn bài toán

 

Bài tương tự: Tìm $p,q$ nguyên tố sao cho $5^p+5^q \vdots pq$

Bài 2: bạn có thể xem lại đề không mình chỉ chứng minh được $n$ có dạng $3^k$ :(

Hình như nó là chứng minh dạng đó, bài 2 ấy, nhưng đề mình bị sai nên ghi lẫn lộn, mình không hiểu nó nói gì nên đăng bài tổng quát lên đây nhờ các cao thủ. Chứng minh được $n=3^{k}$ là tìm được $n$ còn gì nữa. :)) Đấy bạn chứng minh giúp mình luôn được không? :D




#661754 $1+2^{n}+4^{n}$ là số nguyên tố

Đã gửi bởi Dam Uoc Mo on 13-11-2016 - 11:56 trong Số học

Bài 1: Tìm $p,q$ nguyên tố sao cho $2^{p}+2^{q}\vdots (pq).$

Bài 2: Giả sử $1+2^{n}+4^{n}$ là số nguyên tố, $n$ nguyên dương. Tìm $n$ thỏa mãn bài toán.




#661681 $P(x)=a_{2n}x^{2n}+a_{2n-1}x^{2n-1...

Đã gửi bởi Dam Uoc Mo on 12-11-2016 - 22:03 trong Đa thức

Tìm GTNN của $n\in N^{*}$ thỏa mãn tồn tại $P(x)=a_{2n}x^{2n}+a_{2n-1}x^{2n-1}+...+a_{1}x+a_{0 } \in R[x]$ có nghiệm thực và $a_{i}\in [2014;2015]$ với mọi $i=0;1;..;2n$.

P/S: Nhân đây nếu ai có tài liệu bài tập kèm lời giải về đa thức cho em (mình) xin với. Mình cảm ơn.




#659790 CMR: $f([p/3]+1);...;f(p-2)\in \wp$

Đã gửi bởi Dam Uoc Mo on 29-10-2016 - 11:14 trong Số học

Cho:

$f(x)=x^{2}+x+p (p\in N^{*});\\ f(0),f(1),...,f([\frac{p}{3}])\in \wp .$

CMR: $f([p/3]+1);...;f(p-2)\in \wp$




#659506 CMR: $A'A_{1};B'B_{1};C'M$ đồng quy

Đã gửi bởi Dam Uoc Mo on 26-10-2016 - 23:16 trong Hình học

Cho tam giác $ABC$ nội tiếp $(\omega )$. $I$ là tâm nội tiếp $\Delta ABC$. $AI,BI,CI$ cắt $(\omega )$ ở $A',B',C'$. M thuộc cạnh $AB$. Đường qua M song song $AI$ cắt đường qua B vuông góc $BI$ ở $A_{1}$. Đường qua M song song $BI$ cắt đường qua A vuông góc $AI$ ở $B_{1}$.

CMR: $A'A_{1};B'B_{1};C'M$ đồng quy.




#659504 Cho số nguyên dương $a, n$ sao cho tất cả các ước nguyên tố của...

Đã gửi bởi Dam Uoc Mo on 26-10-2016 - 23:09 trong Số học

Cái này chứng minh sao nhỉ $\frac{n-n/p^{s}}{p-1}\leq [ \frac{n}{p-1} ]$ ?

Chỗ đó có gì đâu.

$V_{p}(n!)< \frac{n}{p-1}\Rightarrow V_{p}(n!)\leq [\frac{n}{p-1}]$. 




#659333 Cho số nguyên dương $a, n$ sao cho tất cả các ước nguyên tố của...

Đã gửi bởi Dam Uoc Mo on 25-10-2016 - 21:42 trong Số học

Cho số nguyên dương $a, n$ sao cho tất cả các ước nguyên tố của $a$ đều lớn hơn $n.$ Chứng minh rằng $(a-1)(a^{2}-1)...(a^{n-1}-1)$ chia hết cho $n!.$

Gọi $p$ là 1 ước nguyên tố của $n$.

Có $a^{p-1}\equiv 1(modp)$ nên $V_{p}(VT)\geq V_{p}(\prod_{i=1}^{[\frac{n}{p-1}]}(a^{(p-1)i}-1))\geq [\frac{n}{p-1}] \\ V_{p}(n!)=[n/p]+[n/p^{2}]+...[n/p^{s}]\leq n/p+n/p^{2}+...+n/p^{s}=\frac{n-n/p^{s}}{p-1}\leq [\frac{n}{p-1}] \\ --> Q.E.D.\blacksquare$

P.S: Lâu lắm mới làm bài. Mỏi quá. :))




#659234 $f(mf(n))=f(m).n^{2}$

Đã gửi bởi Dam Uoc Mo on 24-10-2016 - 23:19 trong Số học

Giả sử $f:N^{*}\rightarrow N^{*}$ thỏa mãn $f(mf(n))=f(m).n^{2}$.

a, CMR $f(2003)$ là 1 số nguyên tố hoặc bình phương 1 số nguyên tố. ( Mình chỉ cần $f$ nhân tính nữa thôi, nên ai giải giúp mình nốt).

b, Xây dựng hàm $f$ thỏa mãn đề.




#655570 $x^{n}-nx+1=0$

Đã gửi bởi Dam Uoc Mo on 25-09-2016 - 23:03 trong Dãy số - Giới hạn

Cho PT: $x^{n}-nx+1=0$. CMR pt có 2 nghiệm $\alpha _{n};\beta _{n}: \\ 0<\alpha _{n}<1<\beta _{n}.$

Tìm $\lim_{n\rightarrow oo}\alpha _{n}, \lim_{n\rightarrow oo}\beta _{n}.$

Bài này giải ra và tìm $\lim_{n\rightarrow oo}\alpha _{n}$ là dễ, nhưng còn về $\beta _{n}$ mình chưa giải được, mình không thấy nó tương tự cái kia. Mình cần giúp 1 lời giải cụ thể. :)




#654386 $2^{n}-1 \vdots [p(p+2)]$

Đã gửi bởi Dam Uoc Mo on 16-09-2016 - 18:22 trong Số học

Tìm $n\in N^{*}$ lẻ min thỏa mãn $\exists p,p+2$ đều là số nguyên tố và $2^{n}-1 \vdots [p(p+2)]$




#653526 $A=\sum x^{3} +8(xy^{2}+yz^{2}+zx^...

Đã gửi bởi Dam Uoc Mo on 09-09-2016 - 23:44 trong Bất đẳng thức - Cực trị

Cho $x,y,z\geq 0;x+y+z=4.$

Tìm max: $A=\sum x^{3} +8(xy^{2}+yz^{2}+zx^{2})$

Bài này mình mò được điểm rơi là $\left ( 0;2;2 \right )$ mà chưa tìm được cách.




#653223 $(2^{n}+1)\vdots n^{2}$

Đã gửi bởi Dam Uoc Mo on 07-09-2016 - 21:56 trong Số học

Mình nghĩ 2 cách giải trên đã có nhầm lẫn, mình sẽ tiếp tục ý tưởng như này: $2^{n}+1\equiv 0(mod n^{2})\\ --> (-8)^{l}\equiv 1(mod 9l^{2})\\ -->(-8)^{l}\equiv 1(modl)$. Đến đây có thể gọi m là ước nguyên tố bé nhất của l rồi dùng bậc của số nguyên để giải quyết nốt. :D




#653181 $(2^{n}+1)\vdots n^{2}$

Đã gửi bởi Dam Uoc Mo on 07-09-2016 - 20:21 trong Số học

Đầu tiên ta nhắc lại bổ đề quen thuộc sau : 
Bổ đề (*) : Với $m$ là số nguyên dương và $gcd(a,m)=gcd(b,m)=1$ thì lúc đó nếu $a^x \equiv b^x \pmod{m} ,a^y \equiv b^y \pmod{m}$ 
Thì $a^{gcd(x,y)} \equiv b^{gcd(x,y)} \pmod{m}$ 
Có thể xem cách chứng minh tại đây 
Bổ đề : Nếu $n|2^n+1$ thì $n=3^q.i$   với $(i,3)=1$
Chứng minh : $n$ lẻ đó là điều ta nhận thấy . Gọi $p$ là ước nguyên tố nhỏ nhất của $n$ ta có từ FLT  : 
$2^{p-1} \equiv 1 \pmod{p}$ và $2^{2n} \equiv 1 \pmod{p}$ suy ra $2^{gcd(p-1,2n)} \equiv 1 \pmod{p}$ 
Mà  dễ thấy $gcd(p-1,2n)=2$ (do $p$ là ước nguyên tố nhỏ nhất của $n$ ) 
Do vậy $p=3$ 
Suy ra $n=3^q.i$ với $(i,3)=1$
Quay về bài toán :Xét $n=1$ thì thỏa với $n>1$ thì 
Bằng LTE ta dễ suy ra được $q=1$ do đó $n=3.i$
Gọi $m$ là ước nguyên tố nhỏ nhất của $i$ khi đó : $m|2^{gcd(m-1,2n)}-1$ mà ta có $gcd(m-1,2n)$ nhận giá trị $2$ hay $6$ nếu là $2$ thì $m=3$ 
đây là điều vô lí . Nếu là $6$ thì ta suy ra $m=7$
từ đó suy ra $7|2^n+1$ ,vô lí vì $2^n+1 \equiv 2 \pmod{7}$ do đó $i=1$ hay $n=3$ 
P/s : Thực ra bài này ko  cần những điều kiện trên vẫn giải được ,bạn có thể tham khảo lời giải và có thể lược bỏ một số ý mà bạn cho là không cần thiết

làm rõ giúp mình đoạn này với. 

 

cái này là đề (IMO 1990 )

theo đề thì n chia hết cho 3 

đặt v3(n)=k Ta có  v3(2n+1) =v3(3) + k =1+k

ma 2n+1 chia hết  cho n2 nên  v3(2n+1)$\geq$ v3(n2) nen 1+k$\geq k$ nen k=1

goi q :ước nguyên tố nhỏ nhất của  x

đặt  n=3x ta có  26x$\equiv 1(mod q)$ chọn l là số nhỏ nhất  sao cho 2l$\equiv 1 ( mod q )$ nên  6x chia hết cho  l 

suy ra l = 3:6 

xét l=3 suy ra q=7 tương tự với l=6 thi 63 chia hết  cho l vì vậy l =7.lúc này sẽ mâu thuẫn vi : 26x+1$\equiv 2(mod 7 )$ va n2  chia hết cho 7 

do n chia hết cho 3 nên n =3 .

vậy :n= 3 thõa yêu cầu bài toán 

làm rõ giúp mình đoạn này với.

Đây đúng là 1 bài toán mở rộng IMO. Mình chứng minh được đến kết quả kia thì bị nghẽn nên nhờ mọi người làm nốt, phần dữ kiện thừa là do mình cung cấp từ những gì mình đã có cho m.n đỡ phải c.m ấy mà. :D




#653124 $(2^{n}+1)\vdots n^{2}$

Đã gửi bởi Dam Uoc Mo on 07-09-2016 - 15:46 trong Số học

Cho $n=3.l (l\in N^{*}),l\equiv 1(mod2),(l,3)=1$.

Tìm các giá trị của $l$ để $(2^{n}+1)\vdots n^{2}$.




#652563 $w_{n}=\frac{x_{1}y_{n}+x_{...

Đã gửi bởi Dam Uoc Mo on 03-09-2016 - 11:49 trong Dãy số - Giới hạn

Lưu ý rằng dãy mà hội tụ về $0$ thì nó bị chặn bởi hằng số $C$ nào đó từ đó

$$w_{n} < C \frac{\sum_{i=1}^{n}y_{i}}{n}$$

Theo định lý Cesaro thì dãy trung bình cộng cũng hội tụ về $0$ nên ta có đpcm . Bài này là bổ đề để chứng minh cho trường hợp thay $0$ bằng số $a$ thì ba dãy $(x_{n}),(y_{n}),(w_{n})$ cùng hội tụ về $a$ .

OK cảm ơn Bằng, thì tớ chứng minh bài tổng quát đến đoạn này thì hơi nghẽn. hôm trước có mò ra thêm 1 cách dùng bất đẳng thức đánh giá nữa.




#651889 $w_{n}=\frac{x_{1}y_{n}+x_{...

Đã gửi bởi Dam Uoc Mo on 29-08-2016 - 21:25 trong Dãy số - Giới hạn

Cho $\left ( x_{n} \right ),\left ( y_{n} \right ):\\ limx_{n}=limy_{n}=0.\\ w_{n}=\frac{x_{1}y_{n}+x_{2}y_{n-1}+...+x_{n-1}y_{2}+x_{n}y_{1}}{n}\\ CMR:\\ limw_{n}=0$